返回列表 发帖

大全-2-18

Lark Manufacturing Company initiated a voluntary Quality Circles program for machine operators. Independent surveys of employee attitudes indicated that the machine operators participating in the program were less satisfied with their work situations after two years of the program’s existence than they were at the program’s start. Obviously, any workers who participate in a Quality Circles program will, as a result, become less satisfied with their jobs.


Each of the following, if true, would weaken the conclusion drawn above EXCEPT:


(A) The second survey occurred during a period of recession when rumors of cutbacks and layoffs at Lark Manufacturing were plentiful.


(B) The surveys also showed that those Lark machine operators who neither participated in Quality Circles nor knew anyone who did so reported the same degree of lessened satisfaction with their work situations as did the Lark machine operators who participated in Quality Circles.


(C) While participating in Quality Circles at Lark Manufacturing, machine operators exhibited two of the primary indicators of improved job satisfaction: increased productivity and decreased absenteeism.


(D) Several workers at Lark Manufacturing who had participated in Quality Circles while employed at other companies reported that, while participating in Quality Circles in their previous companies, their work satisfaction had increased.


(E) The machine operators who participated in Quality Circles reported that, when the program started, they felt that participation might improve their work situations.

答案:E

E是没WEAKEN,但是我考虑了半天,觉得B也没WEAKEN,虽然B看起来很像WEAKEN,就好像结论说A会怎样,选项B说A和B同样都会怎样,这个怎么WEAKEN呢???

收藏 分享

本文的推倒是 B(参加末PROGRAM的人的满意度下降)

-> A (PROGRAM 的原因)

因果型.

(B) The surveys also showed that those Lark machine operators who neither participated in Quality Circles nor knew anyone who did so reported the same degree of lessened satisfaction with their work situations as did the Lark machine operators who participated in Quality Circles.


B削弱了.非B-->A.说明不是A的问题.

(A) The second survey occurred during a period of recession when rumors of cutbacks and layoffs at Lark Manufacturing were plentiful.

是典型的它因-->B,同样说明不是A的问题.

TOP

但是B+题目的两个survey好像说明了任何人都会变得less satisfied,这个是把原文的结论包含在内了呀,我觉得两者没有相悖的地方.

TOP

看了二楼的解释,觉得很清楚,再一反思自己的错误,我发现我在WEAKEN的是文中的这句话“Obviously, any workers who participate in a Quality Circles program will, as a result, become less satisfied with their jobs.”参加QCprogram导致满意度下降

不是说WEAKEN结论么???可是CRACKER就能得出要WEAKEN得是:

B(参加末PROGRAM的人的满意度下降)----> A (PROGRAM 的原因)

请指点一下,否则以后这种题我肯定错一片。。。。

TOP

返回列表

站长推荐 关闭


美国top10 MBA VIP申请服务

自2003年开始提供 MBA 申请服务以来,保持着90% 以上的成功率,其中Top10 MBA服务成功率更是高达95%


查看